Club CIE

13»

Réponses

  • oui gb , c’était un oubli en tapant, mais c'est un problème très délicieux
    Le 😄 Farceur


  • $I_{\alpha}=\displaystyle\int_0^{+\infty} \frac {\ln(x^{\alpha}+1)}{x^{\alpha}+1} dx$ pour ${\alpha}>1$ réel s'exprime simplement avec la fonction digamma, $\psi(x)=\dfrac{\Gamma'(x)}{\Gamma(x)}$.

    Quand $\alpha$ est un nombre rationnel elle s'exprime par une somme finie faisant intervenir les fonctions $\ln$, $\sin$ et $\cos$.

    Dans le cas $\alpha$ entier j'ai déjà donné le résultat dans ce fil qui date de 7 ans.
  • Bonjour

    @Gebrane il y a sûrement une faute de frappe dans ton énoncé.
    Je trouve $\displaystyle I_n=\dfrac{\pi}{\sin (\frac{\pi}{n} )}\int_0 ^1 \dfrac{t^{n-2}}{1+t+\cdots+t^{n-1}} dt $
    Exemple $I_3=\dfrac{\pi \ln(3)}{\sqrt{3}}-\dfrac{\pi ^2}{9}$.
     
  • merci bd faute de frappe corrigée
    Le 😄 Farceur


  • Fdp tu avais partagé la référence qui t'avait appris plein de choses sur le calcul d'intégrales. Peux-tu la rappeler ici ? Merci à toi.
  • bd ou jandri peux-tu rédiger ta solution ( dans l'autre lien Jandri n'a pas donné les détails). Ce fil se veut être formateur ( Moi je tape très lent et je commets beaucoup de fautes de frappes)
    Le 😄 Farceur


  • RLS:

    Je ne me souviens plus. S'agit-il d'un livre?
  • Rebonjour
    Ok mais seulement les étapes !

    1. $ \displaystyle f'(t)=\int_0^{\infty }\frac{n t^{n-1} x^n}{\left(1+x^n\right) \left(1+t^n x^n\right)}dx .$
    2. Mais $ \dfrac{n t^{n-1} x^n}{\left(1+x^n\right) \left(1+t^n x^n\right)}=

    \dfrac{n t^{n-1}}{\left(1-t^n\right) \left(1+t^n x^n\right)}-\dfrac{n t^{n-1}}{\left(1-t^n\right) \left(1+x^n\right)}$
    Ce qui permet de couper en 2 l'intégrale du 1, et dans le premier morceau on effectue le changement $x\mapsto x/t $, pour regrouper les 2 intégrales et obtenir : $
    f'(t)=\int_0^\infty \dfrac{n (t-1) t^{n-2}}{\left(t^n-1\right) \left(1+x^n\right)} dx$
    3.$f'(t)=\dfrac{n (t-1) t^{n-2}}{\left(t^n-1\right) } \Gamma[1/n]\Gamma[1-1/n]/n=\dfrac{ (t-1) t^{n-2}}{\left(t^n-1\right)} \times \dfrac{1}{\sin(\pi/n)}$
    4. D'où $ f(t)=\int_0^t f'(u) du $ puis passage à la limite ($t$ tend vers $1$) pour obtenir l'expression que j'ai donnée.
     
  • Oui en effet Fdp, c'était un livre.
  • RLS:

    Ce livre-là peut-être?

    (tape dans Google: almost impossible integrals sums and series ;) )
  • Sans doute, en tous cas merci énormément !
  • Bd donc tu n 'as pas trouvé la formule de Jandri
    Je partage ce document qui donne les détails des calculs
    Le 😄 Farceur


  • Oui exact. Il faut calculer $J_n=\int_0 ^1 \dfrac{t^{n-2}}{1+t+\cdots+t^{n-1}} dt $
    En décomposant en éléments simples on obtient une formule comme @Jandri et mais difficilement simplifiable .
     
  • Je reviens sur le calcul de $I_{\alpha}=\displaystyle\int_0^{+\infty} \frac {\ln(x^{\alpha}+1)}{x^{\alpha}+1} dx$ pour $\alpha>1$ réel.
    En posant $t=x^{\alpha}$ et $\beta=\dfrac1{\alpha}<1$ on obtient $I_{\alpha}=\beta\displaystyle\int_0^{+\infty} \frac {\ln(1+t)}{1+t}t^{\beta-1}dt$.
    On pose $F(x)=\beta\displaystyle\int_0^{+\infty} \frac {\ln(1+xt)}{1+t}t^{\beta-1}dt$.
    $F'(x)=\beta\displaystyle\int_0^{+\infty} \frac {t^{\beta}}{(1+t)(1+tx)}dt=\dfrac{\beta}{x-1}\displaystyle\int_0^{+\infty}\left( \frac1{1+t}- \frac1{1+tx}\right)t^{\beta-1}dt$.
    En utilisant le résultat classique $\displaystyle\int_0^{+\infty} \frac {t^{\beta-1}}{1+t}dt=\dfrac{\pi}{\sin(\beta\pi)}$ pour $0<\beta<1$ on obtient $F'(x)=\dfrac{\beta\pi}{\sin(\beta\pi)}\dfrac{1-x^{-\beta}}{x-1}$.
    On en déduit $I_{\alpha}=F(1)-F(0)=\dfrac{\beta\pi}{\sin(\beta\pi)}J_{\beta}$ avec $J_{\beta}=\displaystyle\int_0^1\dfrac{1-x^{-\beta}}{x-1}dx$.

    Enfin $J_{\beta}=\displaystyle\int_0^1(x^{-\beta}-1)\displaystyle\sum_{k=0}^{+\infty}x^kdx=\displaystyle\sum_{k=0}^{+\infty}\left( \frac1{k+1-\beta}- \frac1{k+1}\right)=-(\gamma+\psi(1-\beta))$ avec $\psi=\dfrac{\Gamma'}{\Gamma}$.

    Les valeurs de $\psi(x)$ pour $x$ rationnel peuvent s'exprimer avec une formule de Gauss (voir Wikipedia) :
    pour $0<p<q$ entiers , ${\displaystyle \psi \left({\frac {p}{q}}\right)+\gamma= -\ln(2q)-{\frac {\pi }{2}}\cot {\frac {p\pi }{q}}+2\sum _{k=1}^{\lfloor {\frac {q-1}{2}}\rfloor }\cos {\frac {2 kp\pi}{q}}\ln \left(\sin {\frac {k\pi }{q}}\right)}$
  • Merci Jandri
    Dans le lien que j'ai partagé, on détaille le calcul sans le besoin des fonctions spéciales
    Le 😄 Farceur


  • Bonjour, je reviens sur $I = \displaystyle \int_0^{+\infty} \dfrac{\ln (1+x^2)}{1+x^2} \mathrm dx$. Comme indiqué par d'autres en divers endroits, considérer $F(t) = \displaystyle \int_0^{+\infty} \dfrac{\ln (1+t^2x^2)}{1+x^2} \mathrm dx$ simplifie considérablement l'évaluation par insertion d'un paramètre de $I$ par la méthode autrefois indiquée sur ce forum par Oumpahpah.
    On note qu'à priori, on a $F(0)=0$ et $F(1)=I$.

    On a:

    1) $I$ est convergente car: $f : x \to \dfrac{\ln (1+x^2)}{1+x^2}$ est continue sur $\mathbb{R^+}$ et $f(x) \sim 2 \dfrac{\ln (x)}{x^2}$ en $+\infty$; par ailleurs, on peut écrire $x^2 = x\sqrt{x}^2$ et $f(x) = \displaystyle 2 \dfrac{ \frac{\ln (x) }{\sqrt{x} }}{x\sqrt{x}}$ et on sait que $ \frac{\ln (x) }{\sqrt{x} } = o(1)$ en $+ \infty$; par suite $f(x) = o\left(x^{-3/2} \right) $en $+ \infty$ et l'intégrale converge par comparaison avec une intégrale de Riemann.

    2) la dérivation sous le signe d'intégrale par $F'(t) = \displaystyle \int_0^{+\infty} \frac{\partial f(t,x)}{\partial t} \mathrm dx$ est justifiée via le théorème de convergence dominée.

    3) en dérivant, on obtient: $\frac{\partial f(t,x)}{\partial t} = 2t \displaystyle \int_0^{+\infty} \dfrac{x^2}{(1+x^2)(1+t^2x^2)} \mathrm dx$.



    4) on utilise deux transformations simples sur les fractions rationnelles:

    i) $\displaystyle \dfrac{u}{1+u}= 1- \dfrac{u}{1+u}$.

    ii) $\displaystyle \dfrac{1}{(1+u)(1+au)} = \displaystyle \dfrac{1}{(1-a)(1+u)} - \dfrac{a}{(1-a)(1+au)} \; \; (*) $ qui se vérifie par réduction au même dénominateur et s'obtient par les techniques habituelles de décomposition des fraction rationnelles.

    5) on a besoin dans la suite d'évaluer:

    i) $ \int_0^{+\infty} \dfrac{1}{1+x^2} \mathrm dx = \displaystyle \lim_{x\to +\infty} \arctan (x) = \pi/2$.

    ii) $ \int_0^{+\infty} \dfrac{1}{1+t^2x^2} \mathrm dx $ par le changement de variable $u=tx$, ,les nouvelles bornes étant $u(0) = 0$ et $\lim_{x\to +\infty} u(x) = +\infty$, avec $\mathrm du = t\mathrm dx $, soit $ \int_0^{+\infty} \dfrac{1}{1+t^2x^2} \mathrm dx = \frac{\pi}{2t} $.

    6) l'application de la formule de dérivation donne en remplaçant $a$ par $t^2$ dans (*) : si $t >0$, $F'(t) =2t \left( \dfrac{\pi}{2t} - \dfrac{\pi}{2} \dfrac{t}{1-t^2} + \dfrac{\pi}{2} \dfrac{1}{1-t^2}\right) = \dfrac{\pi}{1+t} $.

    7) On utilise le théorème fondamental du calcul intégral: si $u \geq 0$, $F(u) = \displaystyle \int_0^u \dfrac{\pi}{1+t} \mathrm dt$, soit $I = F(1) - F(0) = \displaystyle \int_0^1 \dfrac{\pi}{1+t} \mathrm dt = \pi \ln (2)$.
    A demon  wind propelled me east of the sun
  • On peut aussi penser exécuter le changement de variable $ \theta=\arctan t$ qui conduit à : $I =-2 \displaystyle \int_0^{\frac{\pi}2} \ln \cos \theta d\theta$.
  • Ceci étant, on obtient un calcul raisonnable de $ J = \displaystyle \int_0^1 \dfrac{\ln(1+x^2)}{1+x^2} \mathrm dx$ qui est une intégrale définie. On effectue le changement de variable $u= 1/x$ qui envoie 0 sur $+\infty$ et conserve la borne 1 avec $ \mathrm dx = - \frac{\mathrm du}{u^2}$ et donne: $J = - \displaystyle \int_{+\infty}^1 \dfrac{\ln(1+\frac{1}{u^2})}{1+\frac{1}{u^2}} \frac{\mathrm du}{u^2} = \displaystyle \int_1^{+\infty} \dfrac{\ln(1+x^2)}{1+x^2} \mathrm dx - 2\displaystyle \int_1^{+\infty} \dfrac{\ln(x)}{1+x^2} \mathrm dx$. Maintenant, on écrit $K = \displaystyle \int_1^{+\infty} \dfrac{\ln(1+x^2)}{1+x^2} \mathrm dx$ et on a $J + K = \pi \ln(2)$.

    Le calcul précédent montre aussi que $J-K = -2G$ avec $G = \displaystyle \sum_{n=0}^{+\infty} \dfrac{(-1)^n}{(2n+1)^2}$ la constante de Catalan chère à Fdp.

    En effet, le changement de variable $u= 1/x$ dans $ \displaystyle \int_1^{+\infty} \dfrac{\ln(x)}{1+x^2} \mathrm dx$ donne $ \displaystyle \int_1^{+\infty} \dfrac{\ln(x)}{1+x^2} \mathrm dx = - \displaystyle \int_0^1 \dfrac{\ln(x)}{1+x^2} \mathrm dx$. Cette dernière intégrale converge en zéro car la fonction à intégrer est équivalente (en zéro) à $-\ln(x)$ dont on connaît une primitive simple: $ -x\ln(x) + x$ qui admet une limite nulle en zéro du fait que $ \displaystyle \lim_{x\to 0} x\ln (x) = 0$.
    Ensuite, une intégration par parties en posant $U'(x) = \dfrac{1}{1+x^2}$ et $V(x) = \ln (x)$, soit $U(x) = \arctan(x)$ et $V'(x) = 1/x$ mène à $ \displaystyle \int_0^1 \dfrac{\ln(x)}{1+x^2} \mathrm dx = \displaystyle \bigl[ \arctan(x) \ln(x) \bigr]_0^1 - \displaystyle \int_0^1 \dfrac{\arctan(x)}{x} \mathrm dx $.

    on utilise le DSE de $f(x) = \dfrac{\arctan(x)}{x}$ en zéro, soit $ \displaystyle \dfrac{\arctan(x)}{x} = \displaystyle \sum_{n=0}^{+\infty} \dfrac{(-1)^nx^{2n}}{2n+1}$ que l'on intègre terme à terme entre 0 et 1 pour retrouver $G$.

    Bilan: on obtient les deux relations $J+K = \pi \ln (2)$ et $J-K = -2G$, ce qui donne $J =\frac{\pi}{2} \ln (2) - G$, comme l'indiquent Gröbner et Hofreiter dans leur Integraltafel, Zweiter Band (Bestimmte Integrale) chez Springer (1958).
    A demon  wind propelled me east of the sun
  • @Chaurien: j'ai repris la méthode indiquée plus haut...pour me conformer au cahier des charges du fil.
    A demon  wind propelled me east of the sun
  • gilles benson merci pour les détails, c'est très formatrice
    Est ce que tu as vu le lien que j'ai indiqué ?
    Le 😄 Farceur


  • @gebrane: tu veux dire le document que tu as proposé au téléchargement. Je l'ai regardé et j'ai eu du mal avec la typographie mais ce n'est pas perdu.
    A demon  wind propelled me east of the sun
  • Ok merci
    Le 😄 Farceur


  • Une autre façon de calculer $\displaystyle J=\int_0^1 \dfrac{\ln(1+x^2)}{1+x^2}dx$

    \begin{align}K&=\int_0^1 \frac{\ln(1-x^2)}{1+x^2}dx\\
    K-J&\overset{x=\sqrt{\frac{1-\sin t}{1+\sin t}}}=\frac{1}{2}\int_0^{\frac{\pi}{2}}\ln\left(\sin t\right)dt\\
    &=-\frac{1}{4}\pi\ln 2\\
    K&\overset{y=\frac{1-x}{1+x}}=2\int_0^1 \frac{\ln 2}{1+y^2}dy+\int_0^1 \frac{\ln y}{1+y^2}dy-2\int_0^1 \frac{\ln(1+y)}{1+y^2}dy\\
    &=\frac{1}{2}\pi\ln 2-\text{G}-2\int_0^1 \frac{\ln(1+y)}{1+y^2}dy\\
    L&=\int_0^1 \frac{\ln(1+y)}{1+y^2}dy\\
    L&\overset{u=\frac{1-y}{1+y}}=\int_0^1 \frac{\ln\left(\frac{2}{1+u}\right)}{1+u^2}du\\
    &=\int_0^1 \frac{\ln 2}{1+u^2}du-L\\
    L&=\frac{1}{8}\pi\ln 2\\
    K&=\frac{1}{4}\pi\ln 2-\text{G}\\
    \end{align}
    Donc, \begin{align}\boxed{\displaystyle J=\frac{1}{2}\pi\ln 2-\text{G} }\end{align}
    NB: J'admets que:
    \begin{align}\int_0^1 \frac{\ln x}{1+x^2}dx&=-\text{G}\\
    \int_0^{\frac{\pi}{2}}\ln\left(\sin x\right)dx&=-\frac{1}{2}\pi\ln 2
    \end{align}
  • Et maintenant, le gros morceau déjà dépecé par d'autres, soit $I_n = \displaystyle \int_0^{+\infty} \dfrac{\left( \ln(1+x^n\right) }{1+x^n} \mathrm dx $ si $n \geq 2$. La méthode précédente s'applique, donc je ne fournis pas les détails.

    On part de $F(t) = \displaystyle \int_0^{+\infty} \dfrac{\left( \ln(1+t^nx^n\right) }{1+x^n} \mathrm dx $ soit $\dfrac{\partial f(t,x)}{\partial t} = \dfrac{nt^{n-1} x^n }{\left( 1+t^nx^n \right)\left(1 + x^n\right) } = nt^{n-1} \left( \dfrac{1}{1+t^nx^n } - \dfrac{1 }{\left( 1+t^nx^n \right)\left(1 + x^n\right) } \right) $.

    On applique la transformation (*) du post précédent:

    $$\dfrac{\partial f(t,x)}{\partial t} = nt^{n-1} \left( \dfrac{1}{1+t^nx^n } - \dfrac{1 }{\left( 1-t^n \right)\left(1 + x^n\right) } + \dfrac{t^n }{\left( 1-t^n \right)\left(1 + t^nx^n\right) } \right)$$

    On est confronté au calcul de $J_n = \displaystyle \int_0^{+\infty} \dfrac{1 }{1+x^n} \mathrm dx $, ce que je propose de faire par un calcul de résidus en utilisant le contour $C_R = L_1 \cup C_2 \cup L_3$ défini par:

    * $L_1 $ est le segment $[0; \; R]$ avec $R > 0$.


    * $C_2$ est l'arc de cercle défini par: $ t\in [0; \; 1] \to c(t) = Re^{2i\pi t/n}$.
    *$L_3$ est le segment paramétré par $ t \in [0; \; R] \to \ell (t) = (R-t)e^{2i\pi /n}$.

    La fonction $g(z) = \dfrac{1 }{1+z^n} $ admet un seul pôle simple à l'intérieur de ce lacet, soit $z_1 = e^{i\pi /n}$ et on peut appliquer la formule des résidus à $g$:

    $$ \displaystyle \int_{C_R} g(z) \mathrm dz = 2\pi i \; Res (g, z_1)$$

    Je note $\zeta = e^{2\pi i / n}$ et on a $ \displaystyle \prod_{j=0}^{n-1} \left( z - \zeta ^j\right) = z^n - 1$, soit $ \displaystyle \prod_{j=1}^{n-1} \left( z - \zeta ^j\right) = \sum_{k=0}^{n-1} z^k$, ce qui entraîne $ \displaystyle \prod_{j=1}^{n-1} \left( 1 - \zeta ^j\right) = n$ (**). On recherche le résidu pour un pôle simple. les racines de $1+z^n$ sont les $z_k = z_1 \zeta^k$ pour $k \in [0; \; n-1]$ (l'intervalle d'entiers...), soit:

    $$ Res (g, z_1) = \dfrac{1}{\prod_{j=1}^{n-1} \left( z_1 - z_1\zeta ^j\right)}$$

    On évalue donc $P_n = \displaystyle \prod_{j=1}^{n-1} \left( z_1 - z_1\zeta ^j\right) \; = \; z_1 ^{n-1} \prod_{j=1}^{n-1} \left( 1 - \zeta ^j\right)$ sachant que $ z_1 ^n = -1$ et en se rappelant (**), ce qui donne $ -z_1/n$.

    On peut maintenant évaluer l'intégrale curviligne suivant chacune de ses trois composantes:

    1) $ \displaystyle \int_{L_1} g(z) \mathrm dz = \displaystyle \int_0^{R} \dfrac{1}{1+x^n} \mathrm dx $

    2) $ \displaystyle \int_{C_2} g(z) \mathrm dz = o \left(R^{1-n}\right)$ à l'infini (on va faire tendre $R$ vers l'infini...). Ceci se prouve en majorant convenablement le module de l'intégrale.

    3) $ \displaystyle \int_{L_3} g(z) \mathrm dz \; = \; \displaystyle \int_0^{R} \dfrac{1}{1+\left((R- t) e^{2\pi i /n}\right) ^n }e^{2\pi i /n} \mathrm dt \; = \; -e^{2\pi i /n} \displaystyle \int_0^{R} \dfrac{1}{1+x^n} \mathrm dx $

    Bilan: $\displaystyle \lim_{R\to +\infty} \displaystyle \int_{C_R} g(z) \mathrm dz \; = \; J_n \left( 1 - e^{2\pi i /n}\right) \; = \; -2\pi i z_1/n$. Un peu de trigonométrie mène à $J_n = \dfrac{\pi}{n \sin (\pi/n)}$.

    On peut revenir au calcul de $I_n$ ou du moins à l'évaluation de $F'(t)$:

    $$F'(t) \; = \; nt^{n-1} \left( \displaystyle \int_0^{+\infty} \dfrac{1}{1+t^nx^n } \mathrm dx - \displaystyle \int_0^{+\infty} \dfrac{1 }{\left( 1-t^n \right)\left(1 + x^n\right) } \mathrm dx + \displaystyle \int_0^{+\infty} \dfrac{t^n }{\left( 1-t^n \right)\left(1 + t^nx^n\right) } \mathrm dx \right)$$

    1) La première intégrale après le changement de variable $u = tx$ donne $J_n/t$.

    2) la seconde donne $\dfrac{J_n}{1-t^n}$.

    3) la dernière donne $ \dfrac{t^{n-1}J_n}{1-t^n}$.

    En substituant, on arrive à:

    $$F'(t) \; = \; \dfrac{\pi t^{n-2}}{\sin \left(\frac{\pi}{n} \right) } \dfrac{1-t}{1-t^n}$$

    Remarque: pour $n=3$, on écrit $ \dfrac{t(1-t)}{1-t^3} = \dfrac{t}{1+t+t^2} = \dfrac{t + 1/2}{1+t+t^2} - \dfrac{1/2}{1+t+t^2} $ pour obtenir une primitive $\Phi (t) = \frac{1}{2} \ln (1+t+t^2) - \frac{1}{\sqrt{3}} \arctan \left( \dfrac{2t+1}{\sqrt{3}}\right) $, soit $I_3 = F(1) = \dfrac{2 \pi}{ \sqrt{3}} \left( \Phi (1) - \Phi(0)\right) = \dfrac{\pi \ln 3}{\sqrt{3}} - \dfrac{\pi ^2}{9}$
    A demon  wind propelled me east of the sun
  • Bonjour, j'ai tenté la décomposition en éléments simples de $g(t) = \dfrac{t^{n-2}}{1+t+\dots + t^{n-1}}$ et je note $C_n = \dfrac{\pi}{n\sin\left( \frac{\pi}{n}\right) }$ et $K_n = nC_n$. On a donc $F'(t) = K_n \; g(t)$.

    1) Les racines du dénominateur sont les $\zeta ^k ; \; 1 \leq k \leq n-1$ avec $\zeta = \exp\left(\frac{2\pi i}{n} \right) $.
    2) $1+t+\dots + t^{n-1} = \displaystyle \prod_{k=1}^{n-1} \left( t - \zeta^k\right) $.
    3) On écrit donc $g(t) = \displaystyle \sum_{k=1}^{n-1} \dfrac{a_k}{t-\zeta^k}$ et on évalue chaque $a_k$ en considérant :
    $$
    a_k \; = \; \lim_{t\to\zeta^k} \left( t - \zeta^k\right) g(t) \; = \; \dfrac{\zeta^{k(n-2)}}{\displaystyle \prod_{1\leq j \leq n-1 \text{ et } j \neq k } \left( \zeta^k - \zeta^j\right)},

    $$ 4) on factorise par $\zeta^k$ dans chacun des $n-2$ termes du produit pour avoir :
    $$
    a_k \; = \; \dfrac{1}{\displaystyle \prod_{1\leq j \leq n-1 \text{ et } j \neq k } \left( 1- \zeta^{j-k}\right)}.



    $$ Le produit vaut $ \dfrac{\displaystyle \prod_{1\leq j \leq n-1 } \left( 1- \zeta^{j-k}\right)}{1- \zeta^{n-k}} = \dfrac{n}{1- \zeta^{n-k}}$, ce qui fournit la valeur de chaque $a_k$ et la formule :
    $$
    g(t) \; = \; \frac{1}{n} \displaystyle \sum_{k=1}^{n-1} \dfrac{1- \zeta^{n-k}}{t-\zeta^k}.

    $$ Pour continuer, il faut regrouper les racines conjuguées en tenant compte de la parité de $n$ puisque si $n$ est pair, -1 est racine de l'unité. Si $n$ est pair, la somme contient un terme médian qui est $\frac{2}{t+1}$.
    5) cas $n = 2p+1$: la somme contient $2p$ termes, ce qui donne: $ \displaystyle g(t) = \frac{1}{n}\sum_{k=1}^{p} 2 \Re \Big( \dfrac{1- \zeta^{n-k}}{t-\zeta^k} \Big) $.
    Je note $ \theta_k = 2\pi/n$ et $P_k = t^2 - 2 \cos ( \theta_k)t +1$; le calcul de la partie réelle donne :
    $$
    g(t) = \frac{2}{n} \displaystyle \sum_{k=1}^{p} \dfrac{t\left(1-\cos( \theta_k) \right) + 2\cos( 2\theta_k) - \cos( \theta_k))}{P_k} .

    $$ Pour simplifier cette expression, on va jouer sur les lignes trigonométriques de l'angle double :
    i) $1 - \cos( \theta_k) = 2\sin^2( \theta_k/2)$.
    ii) $1 + \cos( \theta_k) = 2\cos^2( \theta_k/2)$.
    iii) $ \sin(\theta_k) = 2\sin( \theta_k/2)\cos( \theta_k/2)$.
    on obtient alors :
    $$
    g(t) = \frac{4}{n} \displaystyle \sum_{k=1}^{p} \dfrac{\sin^2(\theta_k/2)\left(t - 2\cos( \theta_k)-1 \right) }{P_k}.

    $$ On voit que $P'_k = 2(t-\cos(\theta_k)) $ et $P_k = \sin^2(\theta_k) \bigg( \Big( \dfrac{t-\cos(\theta_k) }{\sin(\theta_k)} \Big) ^2+1 \bigg) $ (forme canonique). Ceci permet d'écrire une primitive de $g$, soit $G$ :
    $$
    G(t) = \frac{4}{n} \displaystyle \sum_{k=1}^{p} \sin^2(\theta_k/2) \bigg( \frac{\ln(P_k)}{2} - \cot(\theta_k/2) \arctan\Big( \dfrac{t-\cos(\theta_k) }{\sin(\theta_k)} \Big) \bigg) . \tag{1}

    $$ 6) cas $n = 2p$: il suffit de modifier les bornes de la somme et d'ajouter le terme manquant :
    $$
    G(t) = \frac{2}{n}\ln(t+1) + \frac{4}{n} \sum_{k=1}^{p-1} \sin^2(\theta_k/2) \bigg( \frac{\ln(P_k)}{2} - \cot(\theta_k/2) \arctan\Big( \dfrac{t-\cos(\theta_k) }{\sin(\theta_k)} \Big) \bigg) . \tag{2}

    $$ 7) On peut maintenant donner une formule pour $I_n = \displaystyle \int_0^{+\infty} \dfrac{\ln (1 + x^n}{1+x^n} \mathrm dx$ :
    a) si $n = 2p+1$:
    $$I_n = K_{2p+1} \left( G(1) - G(0) \right) = 4 C_{2p+1} \displaystyle \sum_{k=1}^{p} \sin^2(\theta_k/2) \left(\ln ( 2\sin(\theta_k/2)- \cot( \theta_k/2) \dfrac{\pi - \theta_k}{2} \right) .

    $$ b) si $n = 2p$ :
    $$I_n = K_{2p} \left( G(1) - G(0) \right) = 2C_{2p}\ln(2) + 4 C_{2p} \displaystyle \sum_{k=1}^{p-1} \sin^2(\theta_k/2) \left(\ln ( 2\sin(\theta_k/2)- \cot( \theta_k/2) \dfrac{\pi - \theta_k}{2} \right) .

    $$ On retrouve (sauf erreur) $I_4 = \frac{3\pi \ln 2}{\sqrt{2}} - \frac{\pi^2}{4\sqrt{2}} \; =\;\frac{3\sqrt{2}\pi \ln 2}{2} - \frac{\pi^2 \sqrt{2}}{8} $.
    A demon  wind propelled me east of the sun
  • gilles benson (tu)
    Le 😄 Farceur


  • bonjour Gebrane, je me suis aperçu à postériori que le calcul de résidus que j'ai employé était entièrement décrit dans le livre des Queffelec sur l'analyse complexe chez Calvage et Mounet; j'avais le souvenir d'avoir vu l'emploi d'un secteur angulaire quelque part mais je ne savais plus où. Au départ j'avais feuilleté Cartan mais il ne proposait que le calcul de $\int_{-\infty}^{+\infty} (1+x^{2n})^{-1} \mathrm dx$ sur un demi-cercle et ceci ne fonctionne pas pour les exposants impairs. J'en profite pour signaler le livre de Michel Hervé: Les Fonctions Analytiques aux PUF qui contient des explications détaillées sur l'utilisation du logarithme complexe et son utilisation dans le calcul des résidus.

    https://www.imj-prg.fr/acg/in-memoriam-michel-herve/
    A demon  wind propelled me east of the sun
  • Merci je vais me procurer son livre
    Le 😄 Farceur


Connectez-vous ou Inscrivez-vous pour répondre.